Given an external gear pair where N1 = 20, N2 = 30, determine the distance between two gears centers, c, assuming that the circular pitch for the drive gear (N = 20) is pe=0.26. Ny=30 DRIVEN Ny=20 DRIVE

Answers

Answer 1

The distance between the centers of the two gears, c, is approximately 2.066 units. This takes into account the number of teeth and the circular pitch for the drive gear in the external gear pair, ensuring proper engagement and operation of the gears.

In an external gear pair, the distance between the gear centers, c, can be calculated using the circular pitch and the number of teeth on both the drive and driven gears.

Given the information provided:
- Drive gear (N1) has 20 teeth
- Driven gear (N2) has 30 teeth
- Circular pitch for the drive gear (pe) is 0.26

To determine the distance between the gear centers, we can use the formula:

c = (N1 + N2) * pe / (2 * π)

Plugging in the given values:

c = (20 + 30) * 0.26 / (2 * π) = 50 * 0.26 / (2 * π) ≈ 2.066

To know more about gear pair visit:

https://brainly.com/question/31482572

#SPJ11


Related Questions

Two sites are being considered for wind power generation. On the first site, the wind blows steadily at 7 m/s for 3000 hours per year. On the second site, the wind blows steadily at 10 m/s for 2000 hours per year. The density of air on the both sites is 1.25 kg/m3 . Assuming the wind power generation is negligible during other times.Calculate the maximum power of wind on each site per unit area, in kW/m2 .

Answers

Solution :

Given :

[tex]$V_1 = 7 \ m/s$[/tex]

Operation time, [tex]$T_1$[/tex] = 3000 hours per year

[tex]$V_2 = 10 \ m/s$[/tex]

Operation time, [tex]$T_2$[/tex] = 2000 hours per year

The density, ρ = [tex]$1.25 \ kg/m^3$[/tex]

The wind blows steadily. So, the K.E. = [tex]$(0.5 \dot{m} V^2)$[/tex]

                                                             [tex]$= \dot{m} \times 0.5 V^2$[/tex]

The power generation is the time rate of the kinetic energy which can be calculated as follows:

Power = [tex]$\Delta \ \dot{K.E.} = \dot{m} \frac{V^2}{2}$[/tex]

Regarding that [tex]$\dot m \propto V$[/tex]. Then,

Power [tex]$ \propto V^3$[/tex] → Power = constant x [tex]$V^3$[/tex]

Since, [tex]$\rho_a$[/tex] is constant for both the sites and the area is the same as same winf turbine is used.

For the first site,

Power, [tex]$P_1= \text{const.} \times V_1^3$[/tex]

            [tex]$P_1 = \text{const.} \times 343 \ W$[/tex]

For the second site,

Power, [tex]$P_2 = \text{const.} \times V_2^3 \ W$[/tex]

           [tex]$P_2 = \text{const.} \times 1000 \ W$[/tex]

who was part of dempwolf his firm when he first started

Answers

Explanation:

Dempwolf created by John Augustus, Among the most prominent innovative solutions in Southern California Pennsylvania was established by Dempwolf with  brother Reinhardt or uncle's son Frederick entered the company of J.A. Dozens of structures in 10 states were engineered by Dempwolf.

A group of students launches a model rocket in the vertical direction. Based on tracking data, they determine that the altitude of the rocket was 89.6 ft at the end of the powered portion of the flight and that the rocket landed 16.5 s later. The descent parachute failed to deploy so that the rocket fell freely to the ground after reaching its maximum altitude. Assume that g = 32.2 ft/s2.
Determine
(a) the speed v1 of the rocket at the end of powered flight,
(b) the maximum altitude reached by the rocket.

Answers

Answer:

[tex]u = 260.22m/s[/tex]

[tex]S_{max} = 1141.07ft[/tex]

Explanation:

Given

[tex]S_0 = 89.6ft[/tex] --- Initial altitude

[tex]S_{16.5} = 0ft[/tex] -- Altitude after 16.5 seconds

[tex]a = -g = -32.2ft/s^2[/tex] --- Acceleration (It is negative because it is an upward movement i.e. against gravity)

Solving (a): Final Speed of the rocket

To do this, we make use of:

[tex]S = ut + \frac{1}{2}at^2[/tex]

The final altitude after 16.5 seconds is represented as:

[tex]S_{16.5} = S_0 + ut + \frac{1}{2}at^2[/tex]

Substitute the following values:

[tex]S_0 = 89.6ft[/tex]       [tex]S_{16.5} = 0ft[/tex]     [tex]a = -g = -32.2ft/s^2[/tex]    and [tex]t = 16.5[/tex]

So, we have:

[tex]0 = 89.6 + u * 16.5 - \frac{1}{2} * 32.2 * 16.5^2[/tex]

[tex]0 = 89.6 + u * 16.5 - \frac{1}{2} * 8766.45[/tex]

[tex]0 = 89.6 + 16.5u- 4383.225[/tex]

Collect Like Terms

[tex]16.5u = -89.6 +4383.225[/tex]

[tex]16.5u = 4293.625[/tex]

Make u the subject

[tex]u = \frac{4293.625}{16.5}[/tex]

[tex]u = 260.21969697[/tex]

[tex]u = 260.22m/s[/tex]

Solving (b): The maximum height attained

First, we calculate the time taken to attain the maximum height.

Using:

[tex]v=u + at[/tex]

At the maximum height:

[tex]v =0[/tex] --- The final velocity

[tex]u = 260.22m/s[/tex]

[tex]a = -g = -32.2ft/s^2[/tex]

So, we have:

[tex]0 = 260.22 - 32.2t[/tex]

Collect Like Terms

[tex]32.2t = 260.22[/tex]

Make t the subject

[tex]t = \frac{260.22}{ 32.2}[/tex]

[tex]t = 8.08s[/tex]

The maximum height is then calculated as:

[tex]S_{max} = S_0 + ut + \frac{1}{2}at^2[/tex]

This gives:

[tex]S_{max} = 89.6 + 260.22 * 8.08 - \frac{1}{2} * 32.2 * 8.08^2[/tex]

[tex]S_{max} = 89.6 + 260.22 * 8.08 - \frac{1}{2} * 2102.22[/tex]

[tex]S_{max} = 89.6 + 260.22 * 8.08 - 1051.11[/tex]

[tex]S_{max} = 1141.0676[/tex]

[tex]S_{max} = 1141.07ft[/tex]

Hence, the maximum height is 1141.07ft

The driver of the truck has an acceleration of 0.4g as the truck passes over the top A of the hump in the road at constant speed. The radius of curvature of the road at the top of the hump is 98 m, and the center of mass G of the driver (considered a particle) is 2 m above the road. Calculate the speed v of the truck.

Answers

Answer:

19.81 m/s

Explanation:

The total acceleration of the truck (a) is due to the centripetal acceleration and as a result of the linear acceleration. Therefore the total acceleration (a) is given by:

[tex]a^2=a_n^2+a_t^2\\\\where\ a_n=centripetal\ acceleration=\frac{v^2}{r},a_t=linear \ acceleration\\\\But\ since\ the \ speed\ is \ constant, the \ linear \ acceleration(a_t)\ would\ be\ 0.\\\\a^2=a_n^2+a_t^2\\\\a^2=a_n^2\\\\a=a_n=\frac{v^2}{r} \\\\v^2=ar\\\\v=\sqrt{ar} \\\\a=0.4g=0.4*9.81,r=98\ m+2\ m=100\ m\\\\v=\sqrt{0.4*9.81*100} \\\\v=19.81\ m/s[/tex]

What current works best when the operator
encounters magnetic arc blow?

•DCEP

•ACEN

•CC

•AC

Answers

Answer:

AC

Explanation:

One situation when alternating current would work better than direct current is if the operator is encountering magnetic arc blow.

Current works best when the operator  encounters magnetic arc blow is AC

Magnetic arc blow is simply defined as the arc deflection due to the warping of the magnetic field that is produced by electric arc current.

This is caused as a result of the following;

- if the material being welded has residual magnetism at an intolerable level

- When the weld root is being made, and the welding current is direct current which indicates constant direction and maintains constant polarity (either positive or negative).

Since it is caused by DC(Direct Current) which means constant polarity , it means the opposite will be better which is AC(alternating current) because it means that electricity direction will be switching to and fro and as such the polarity will also be revered in response to this back and forth switch manner.

Thus, Current works best when the operator  encounters magnetic arc blow is AC

Read more at; brainly.in/question/38789815?tbs_match=1

An example of a power consuming device would be a(n) ____ while an example of a non-power consuming device would be a(n) ____.

Answers

An example of a power consuming device would be a(n) light bulb, a computer device while an example of a non-power consuming device would be a(n) switch or button.

What is power consumption?

The quantity of energy utilized per unit of time is known as power consumption. Power use is a significant factor in digital systems.

Power consumption is a limiting factor for the battery life of portable devices like laptop computers and cell phones.

Learn more about power consumption:
https://brainly.com/question/14860418
#SPJ1

The pressure less than atmospheric pressure is known as:

Suction pressure

Negative gauge pressure

Vacuum pressure

All of the above

Answers

Answer:

answer is option (d) all of the above

A 50-mm cube of the graphite fiber reinforced polymer matrix composite material is subjected to 125-kN uniformly distributed compressive force in the direction 2, which is perpendicular to the fiber direction (direction 1). The cube is constrained against expansion in direction 3. Determine:

a. changes in the 50-mm dimensions.
b. stresses required to provide constraints.

Answers

Answer:

hello some parts of your question is missing attached below is the missing part

answer :

A) Determine changes in the 50-mm dimensions

The changes are : 0.006mm compression  in y-direction

                               0.002 mm expansion in x and z directions

B) the stress required are evenly distributed

Explanation:

Given data :

50-mm cube of graphite fiber reinforced polymer matrix

subjected to 125-KN force in direction 2,

direction 2 is perpendicular to fiber direction ( direction 1 ) and cube is constrained against expansion in direction 3

A) Determine changes in the 50-mm dimensions

The changes are : 0.006mm compression  in y-direction

                               0.002 mm expansion in x and z directions

B) the stress required are evenly distributed

attached below is the detailed solution

This is silence I couldent find the tab... 30 points plus marked brainliest if corrects!


The most recent evidence supporting the theory of plate tectonics would include
es )
A)
GPS monitoring of plate speeds and movements.
B)
the WWII discovery of paleomagnetic reversals.
Elimi
O
the 1963 mapping of the tectonic plate boundaries.
D
C-14 dating of marine fossils found in the Himalayas.

Answers

Yeah the answer is A

I have an AC waveform with the voltage peak value of 100 V. I'm trying to find the RMS value of the voltage​

Answers

Answer:

98 x 100=9,00

Explanation:

What are the top 4 solar inventions, how they are used, and how they are better than the original way of powering them

Answers

Yes I will answer soon

Check Your Understanding: True Stress and Stress A cylindrical specimen of a metal alloy 47.7 mm long and 9.72 mm in diameter is stressed in tension. A true stress of 399 MPa causes the specimen to plastically elongate to a length of 54.4 mm. If it is known that the strain-hardening exponent for this alloy is 0.2, calculate the true stress (in MPa) necessary to plastically elongate a specimen of this same material from a length of 47.7 mm to a length of 57.8 mm.

Answers

Answer:

The answer is "583.042533 MPa".

Explanation:

Solve the following for the real state strain 1:

[tex]\varepsilon_{T}=\In \frac{I_{il}}{I_{01}}[/tex]

Solve the following for the real stress and pressure for the stable.[tex]\sigma_{r1}=K(\varepsilon_{r1})^{n}[/tex]

[tex]K=\frac{\sigma_{r1}}{[\In \frac{I_{il}}{I_{01}}]^n}[/tex]

Solve the following for the true state stress and stress2.

[tex]\sigma_{r2}=K(\varepsilon_{r2})^n[/tex]

     [tex]=\frac{\sigma_{r1}}{[\In \frac{I_{il}}{I_{01}}]^n} \times [\In \frac{I_{i2}}{I_{02}}]^n\\\\=\frac{399 \ MPa}{[In \frac{54.4}{47.7}]^{0.2}} \times [In \frac{57.8}{47.7}]^{0.2}\\\\ =\frac{399 \ MPa}{[ In (1.14046122)]^{0.2}} \times [In (1.21174004)]^{0.2}\\\\ =\frac{399 \ MPa}{[ In (1.02663509)]} \times [In 1.03915873]\\\\=\frac{399 \ MPa}{0.0114161042} \times 0.0166818905\\\\= 399 \ MPa \times 1.46125948\\\\=583.042533\ \ MPa[/tex]

Describe how to contribute to
zero/low carbon work outcomes
within the built environment.

Answers

Answer:

day if you workout without Zero billing that means you're not sweating. Sweating you're not losing anything that means you have zero outcomes

Explanation:

Which of the following choices accurately contrasts a categorical syllogism with a conditional syllogism?


An argument constructed as a categorical syllogism uses deductive reasoning whereas an argument constructed as a conditional syllogism uses inductive reasoning.

A categorical syllogism contains two premise statements and one conclusion whereas a conditional syllogism contains one premise statement and one conclusion.

A categorical syllogism argues that A and B are both members of C whereas a conditional syllogism argues that if A is true then B is also true.

An argument constructed as a categorical syllogism is valid whereas an argument constructed as a conditional syllogism is invalid.

Answers

Answer:

The correct option is - A categorical syllogism argues that A and B are both members of C whereas a conditional syllogism argues that if A is true then B is also true.

Explanation:

As,

Categorical syllogisms follow an "If A is part of C, then B is part of C" logic.

Conditional syllogisms follow an "If A is true, then B is true" pattern of logic.

So,

The correct option is - A categorical syllogism argues that A and B are both members of C whereas a conditional syllogism argues that if A is true then B is also true.

what is the distance in term of wavelengh between successive minima in the standing wave ratio​

Answers

Answer:

hi there

Explanation:

Lab scale tests performed on a cell broth with a viscosity of 5cP gave a specific cake resistance of 1 x1011 cm/g and a negligible medium resistance. The cake solids (dry basis) per volume of filtrate was 20 g/liter. It is desired to operate a larger rotary vacuum filter (diameter 8 m and length 12 m) at a vacuum pressure of 80 kPA with a cake formation time of 20 s and a cycle time of 60 s. Determine the filtration rate in volumes/hr expected for the rotary vacuum filter.

Answers

Answer:

5.118 m^3/hr

Explanation:

Given data:

viscosity of cell broth = 5cP

cake resistance = 1*1011 cm/g

dry basis per volume of filtrate = 20 g/liter

Diameter = 8m ,  Length = 12m

vacuum pressure = 80 kpa

cake formation time = 20 s

cycle time = 60 s

Determine the filtration rate in volumes/hr  expected fir the rotary vacuum filter

attached below is a detailed solution of the question

Hence The filtration rate in volumes/hr expected for the rotary vacuum filter

V' = ( [tex]\frac{60}{20}[/tex] ) * 1706.0670

   = 5118.201 liters  ≈ 5.118 m^3/hr

If the sum of the two numbers is 4 and the sum of their squares minus three times their product is 76,find the number

Answers

Answer:

-2 and 6

Explanation:

Let "x" and "y" be 2 numbers.

The sum of the two numbers is 4. The mathematical expression is:

x + y = 4

y = 4 - x   [1]

The sum of their squares minus three times their product is 76. The mathematical expression is:

x² + y² - 3 x y = 76   [2]

If we substitute [1] in [2], we get:

x² + (4 - x)² - 3 x (4 - x) = 76

x² + 16 - 8 x + x² - 12 x + 3 x² = 76

5 x² - 20 x - 60 = 0

We apply the solving formula for second order equations and we get x₁ = 6 and x₂ = -2.

If we replace these x values in [1], we get:

y₁ = 4 - x₁ = 4 - 6 = -2

y₂ = 4 - x₂ = 4 - (-2) = 6

As a consequence, one of the numbers is 6 and the other is -2.

A sign structure on the NJ Turnpike is to be designed to resist a wind force that produces a moment of 25 k-ft in one direction. The axial load is 30 kips. Soil conditions consist of a normally consolidated clay layer with following properties; su=800 psf, andγsat= 110 pcf. Design for a FOS of 3. Assume frost depth to be 3ft below grade

Answers

Solution :

Finding the cohesion of the soil(c) using the relation:

[tex]$c = \frac{q_u}{2}$[/tex]

Here, [tex]$q_u$[/tex] is the unconfined compression strength of the soil;

[tex]$c = \frac{800}{2}$[/tex]

   = 400 psf

∴ The cohesion value is greater than 0

So the use of the angle of internal friction is 0

Referring to the table relation between bearing capacity factors and angle of internal friction.

For the angle of inter friction [tex]$0^\circ$[/tex]

    [tex]$N_c = 5.14$[/tex]

   [tex]$N_q = 1.0$[/tex]

   [tex]$N_r = 0$[/tex]

Therefore,

[tex]$q_{ult} = (400 \times 5.14 )+(110 \times 3 \times 1.0)+(0.5 \times 100 \times 13 \times 0)$[/tex]

     =  2386 psf

∴ Allowable bearing capacity [tex]$q_{a} = \frac{Q_{allow}}{A}$[/tex]

                                                     [tex]$=\frac{30}{B^2}$[/tex]

∴ [tex]$q_a = \frac{q_{ult}}{F.O.S}$[/tex]

  [tex]$\frac{30}{B^2} = \frac{2386}{3}$[/tex]

∴ B = 0.2 ft

Therefore, the dimension of the square footing is 0.2 ft x 0.2 ft

                                                                                 [tex]$=0.04 \ ft^2$[/tex]

A cylindrical bar of metal having a diameter of 15.7 mm and a length of 178 mm is deformed elastically in tension with a force of 49100 N. Given that the elastic modulus and Poisson's ratio of the metal are 67.1 GPa and 0.34 respectively, Determine the following:(a) The amount by which this specimen will elongate (in mm) in the direction of the applied stress. The entry field with incorrect answer now contains modified data.(b) The change in diameter of the specimen (in mm). Indicate an increase in diameter with a positive number and a decrease with a negative number.

Answers

Answer:

0.6727

-0.02017

Explanation:

diameter = 15.7

l = 178

E =elastic modulus = 67.1 Gpa

poisson ratio = 0.34

p = force = 49100N

first we calculate the area of the cross section

[tex]A=\frac{\pi }{4} d^{2}[/tex]

[tex]A=\frac{\pi }{4} (15.7)^{2} \\A = \frac{774.683}{4} \\[/tex]

A = 193.6mm²

1. Change in directon of the applied stress

[tex]= \frac{pl}{AE}[/tex]

= 49100*178/193.6*67.1*10³

= [tex]=\frac{8739800}{12990560}[/tex]

δl = 0.6727  mm

2. change in diameter of the specimen

equation for poisson distribution =

m = -(δd/d) / (δl/l)

0.34 = (δd/15.7) / (0.6727/178)

0.34 = (-δd * 178) / 15.7 * 0.6727

0.34 = -178δd / 10.56139

we cross multiply

10.56139*0.34 =-178δd

3.5908726 = -178δd

δd = 3.5908726/-178

δd = -0.02017 mm

the change in dimeter has a negative sign. it decreases

Now, you get a turn to practice writing a short program in Scratch. Try to re-create the program that was shown that turns the sprite in a circle. After you have completed that activity, see if you can make one of the improvements suggested. For example, you can try adding a sound. If you run into problems, think about some of the creative problem-solving techniques that were discussed.

When complete, briefly comment on challenges or breakthroughs you encountered while completing the guided practice activity.


Pls help im giving 100 points for this i have this due in minutes

Answers

Answer:

u need to plan it out

Explanation:

u need to plan it out

Answer:

use the turn 1 degrees option and put a repeat loop on it

Explanation:

u can add sound in ur loop

Find the derivative of x ​

Answers

Answer:

this is your answer. if mistake don't mind.

A single crystal of a metal that has the FCC crystal structure is oriented such that a tensile stress is applied parallel to the [100] direction. If the critical resolved shear stress for this material is 2.00 MPa, calculate the magnitude of applied stress necessary to cause slip to occur on the (111) plane in the direction.

Answers

Answer:

Explanation:

From the given information:

The equation for applied stress can be expressed as:

[tex]\sigma_{app} = \dfrac{\tau_{CRSS}}{cos \phi \ cos \lambda}[/tex]

where;

[tex]\phi[/tex] = angle between the applied stress [100] and [111]

To determine the [tex]\phi[/tex] and [tex]\lambda[/tex] for the system

Using the equation:

[tex]\phi= cos^{-1}\Big [\dfrac{l_1l_2+m_1m_2+n_1n_2}{\sqrt{(l_1^2+m_1^2+n_1^2)(l_2^2+m_2^2+n_2^2)}}\Big][/tex]

for [100]

[tex]l_1 = 1, m_1 = 0, n_1 = 0[/tex]

for [111]

[tex]l_1 = 1 , m_1 = 1, n_1 = 1[/tex]

Thus;

[tex]\phi= cos^{-1}\Big [\dfrac{1*1+0*1+0*1}{\sqrt{(1^2+0^2+0^2)(1^2+1^2+1^2)}}\Big][/tex]

[tex]\phi= cos^{-1}\Big [\dfrac{1}{\sqrt{(3)}}\Big][/tex]

[tex]\phi= 54.74^0[/tex]

To determine  [tex]\lambda[/tex]  for [tex][1 \overline 1 0][/tex]

where;

for [100]

[tex]l_1 = 1, m_1 = 0, n_1 = 0[/tex]

for [tex][1 \overline 1 0][/tex]

[tex]l_1 = 1 , m_1 = -1, n_1 = 0[/tex]

Thus;

[tex]\lambda= cos^{-1}\Big [\dfrac{1*1+0*1+0*0}{\sqrt{(1^2+0^2+0^2)(1^2+(-1)^2+0^2)}}\Big][/tex]

[tex]\phi= cos^{-1}\Big [\dfrac{1}{\sqrt{(2)}}\Big][/tex]

[tex]\phi= 45^0[/tex]

Thus, the magnitude of the applied stress can be computed as:

[tex]\sigma_{app} = \dfrac{\tau_{CRSS}}{cos \phi \ cos \lambda }[/tex]

[tex]\sigma_{app} = \dfrac{2.00}{cos (54.74) \ cos (45) }[/tex]

[tex]\mathbf{\sigma_{app} =4.89 \ MPa}[/tex]

An air heater may be fabricated by coiling Nichrome wire and passing air in cross flow over the wire. Consider a heater fabricated from wire of diameter D=1 mm, electrical resistivity rhoe=10−6Ω⋅m, thermal conductivity k=25W/m⋅K, and emissivity ε=0.20. The heater is designed to deliver air at a temperature of T[infinity]=50∘C under flow conditions that provide a convection coefficient of h=250W/m2⋅K for the wire. The temperature of the housing that encloses the wire and through which the air flows is Tsur=50∘C. If the maximum allowable temperature of the wire is Tmax=1200∘C, what is the maximum allowable electric current I? If the maximum available voltage is ΔE=110V, what is the corresponding length L of wire that may be used in the heater and the power rating of the heater? Hint: In your solution, assume negligible temperature variations within the wire, but after obtaining the desired results, assess the validity of this assumption.

Answers

Solution :

Assuming that the wire has an uniform temperature, the equivalent convective heat transfer coefficient is given as :

[tex]$h_T= \epsilon \sigma (T_s+T_{surr})(T_s^2 +T^2_{surr})$[/tex]

[tex]$h_T= 0.20 \times 5.67 \times 10^{-8} (1473+323)(1473^2 +323^2)$[/tex]

[tex]$h_T=46.3 \ W/m^2 .K$[/tex]

The total heat transfer coefficient will be :

[tex]$h_T=(250+46.3) \ W/m^2 .K$[/tex]

    [tex]$=296.3 \ W/m^2 .K$[/tex]

Now calculating the maximum volumetric heat generation :

[tex]$\dot {q}_{max}=\frac{2h_t}{r_0}(T_s-T_{\infty})$[/tex]

[tex]$\dot {q}_{max}=\frac{2\times 296.3}{0.0005}(1200-50)$[/tex]

        [tex]$= 1.362 \times 10^{9} \ W/m^3$[/tex]

The heat generation inside the wire is given as :

[tex]$\dot{q} = \frac{I^2R}{V}$[/tex]

Here, R is the resistance of the wire

         V is the volume of the wire

∴ [tex]$\dot{q} = \frac{I^2\left( \rho \times \frac{L}{A} \right)}{A \times L}$[/tex]

     [tex]$=\frac{I^2 \rho}{\left(\frac{\pi}{4}D^2 \right)}$[/tex]

where, ρ is the resistivity.

[tex]$I_{max}= \left(\frac{\dot{q}_{max}}{\rho} \right)^{1/2} \times \frac{\pi}{4}D^2$[/tex]

[tex]$I_{max}= \left(\frac{1.36 \times 10^9}{10^{-6}} \right)^{1/2} \times \frac{3.14}{4}(1 \times 10^{-3})^2$[/tex]

        = 28.96 A

Now considering the relation for the current flow through the finite potential difference.

[tex]$E=I_{max} \times R$[/tex]

[tex]$E=I_{max} \times \rho \times \frac{L}{A}$[/tex]

[tex]$L=\frac{AE}{I_{max} \ \rho}$[/tex]

[tex]$L=\frac{\frac{\pi}{4} \times (1 \times 10^{-3})^2 \times 110}{28.96 \times 10^{-6}}$[/tex]

   = 2.983 m

Now calculating the power rating of the heater:

[tex]$P= E \times I_{max}$[/tex]

[tex]$P= 110 \times 28.96}$[/tex]

   = 3185.6 W

   = 3.1856 kW

how skateboards works?

Answers

Answer

The skateboarder applies pressure to the trucks and gives/releases pressure on the levers. Second, the wheels and the axles are also examples of simple machines. They help the skater ride, spin, grind, and do a bunch of other radical movements on a skateboard.:

Explanation:

I don’t know the answer to this question

Answers

Answer:

I dont know the answer either

Explanation:

Answer:

flux

Explanation:

With _____, only one criterion must evaluate true in order for a record to be selected and with _____, all criteria must be evaluate true in order for a record to be selected.

a. parameter criteria, double criteria
b. function criteria, IF criteria
c. simple criteria, complex criteria
d. OR criteria, AND criteria

Answers

Answer:

d

Explanation:

OR criteria, AND criteria

In an OR criteria, it doesn't need all the records to be true. Just one record is enough and all other criterion becomes true.

In an AND criteria, it's unlike the OR criteria and works in opposite. It needs every member of the record to be true to be able to adjudge the whole record as true.

And as such, we have

With OR criteria, only one criterion must evaluate true in order for a record to be selected and with AND criteria, all criteria must be evaluate true in order for a record to be selected.

During a shrinkage limit test, a 19.3 cm3 saturated clay sample with a mass of 37 g was placed in a porcelain dish and dried in the oven. The oven-dried sample had a mass of 28 g with a final volume of 16 cm3 . Determine the shrinkage limit and the shrinkage ratio.

Answers

Answer:

shrinkae limit = 20.35%

shrinkage ratio = 1.45

Explanation:

1. to get the shrinkage limit we would first calculate the moisture content w.

w = (37-28)/28

= 9/28

= 0.3214

then the formula for shrinkage limit is

[tex][w-\frac{(V-Vd}{wd} ]*100[/tex]

w = 0.3214

V = 19.3

Vd = 16

Wd = 28

when we put these values into the formula:

[tex][0.3214-\frac{(19.3-16)}{28} ]*100\\[/tex]

= 20.35%

2. the shrinkage limit = Wd/V

= 28/19.3

= 1.45

1. What is the productivity rate using cycle time for the following information:
I
Type of Work – Hauling
Average Cycle Time – 35 Minutes
Truck Capacity – 25 Tons
Crew - One Driver
Productivity Factor - 0.85
System Efficiency – 55 Minutes
per
Hour

Answers

ndjdjdhdjshdhdjdjjdjdjdjdhdhhdhdhdhdhdhdhdhdhdhdh

Tech A says that LED brake lights illuminate faster than incandescent bulbs. Tech B says that LED brake lights have
more visibility and last longer. Who is correct?

Answers

Answer:

Both

Explanation:

If the hypotenuse of a right triangle is 12 and an acute angle is 37 degrees find leg a and leg b lengths

Answers

scrity añao devid codicie

Other Questions
The strategies with the least economies of scale would typically tend to be of which form: Broad differentiation Focused differentiation Broad cost leadership Focused low cost given g(x)=x53x4 2, find the x-coordinates of all local minima using the second derivative test. if there are multiple values, give them separated by commas. if there are no local minima, enter . If the coefficient of the correlation is -0.4,then the slope of the regression line a.must also be -0.4 b.can be either negative or positive c.must be negative d.must be 0.16 Last Friday mega millions prize was estimated to be 145 million. Report lottery prizes are just the sum total of the future annual payment. In this case. 26 beginners of th year annuity payments that will be paid to the prize Winner. For the current prize:annual payment =145 million/26. Of course,this145 million annuity ignores the time value of money.The winner can elect the cash option of98 million today instead of 26 year annuity. Whats the difference between today pv of the cash option and annuity payout atw3.5% compound annuallyA -47milliknB -3.803C-4.321D0.506E3.803 let f(x,y,z)=5z2xi (53y3 tan(z))j (5x2z 5y2)k. use the divergence theorem to evaluate sf ds where s is the top half of the sphere x2 y2 z2=1 oriented upwards. T/F the domestication of plants and animals occurred for food independently occured in both the old world and the americas 11,000 years ago Complete each sentence by selecting the correct awnser from the drop-down menu. The first five stanzas of a villanelle have (four, three, two) lines eachThe final stanza of a villanelle has (four, three, two) lines Repetition in a villanelle is used for (distraction, uncertainty, emphasis) Speed A cart, weighing 24.5 N, is released from rest on a 1.00-m ramp, inclined at an angle of 30.0 as shown in Figure 16. The cart rolls down the incline and strikes a second cart weighing 36.8 N.a. Define the two carts as the system. Calculate the speed of the first cart at the bottom of the incline.b. If the two carts stick together, with what initial speed will they move along? compute the second-order partial derivative of the function (,)=/ 25. Evaluate the function as indicated. Use a calculator only if it is necessary or more efficient. (Round your answers to three decimal places. )G(-1) = 4. 4x which type of business would be most likely to use a job order costing system Explain why the relation R on {0, 1, 2} given byR = {(0, 0), (1, 1), (2, 2), (0, 1), (1, 0), (1, 2), (2, 1)}is not an equivalence relation. Be specific. you are given the parametric equations x=te^t,\;\;y=te^{-t}. (a) use calculus to find the cartesian coordinates of the highest point on the parametric curve. A telephone company offers a monthly cellular phone plan for $19.99. It includes 250 anytime minutes plus $0.25 per minute for additional minutes. The following function is used to compute the monthly cost for a subscriber, where x is the number of anytime minutes used 19.99 if 0250 Compute the monthly cost of the cellular phone for use of the following anytime minutes. (b) 280 (c) 251 (a) 115 I was afraid the friends of the deceased, made sore by the loss of their principal, wouldagain blow up the embers of party & dissention, and disturb the harmony & vigour of theCivil & military authorities. Which sentence from "Brothers of the Revolution" gives an example of the concernsWalton expresses in the excerpt from his letter to Washington? what are the top 10 questions to ask an interviewer a taxpayer has the following capital gains and losses: ltcg $15,000, ltcl (8,000); stcg $11,000, and stcl ($16,000). what are the net capital gain or loss and the character? Incandescent lightbulbshave a skinny wire in themiddle called a filament. Why is the wire in themiddle so skinny?A. A skinny wire reduces the resistance. B. It increases friction, which increases heat, which makeslight. C. The skinny wire increases the conductivity. Consider the following data:12,9,7,8,5,1Step 1 of 3: Calculate the value of the sample variance. Round your answer to one decimal place.Step 2 of 3: Calculate the value of the sample standard deviation. Round your answer to one decimal place.Step 3 of 3: Calculate the value of the range. You are flying a kite at the beach on a hot summer aftemoon. The kite will blow toward the O a. land in both hemispheres. O b. land in the southern hemisphere and ocean in the northern hemisphere. O c ocean in both hemispheres. O d. land in the northern hemisphere and ocean in the southern hemisphere.